part of the 20 kilometres was in a road and the rest was on a footpath
The ratio
road distance:footpath distance 3:2
work out the road distance

Answers

Answer 1

The distance of road is 12 kilometres

What is Ratio?

Ratio is  shows how many times one number contains another. It is also the comparison of two quantities of the same units that indicates how much of one quantity is present in the other quantity

How to determine this

When total distance = 20 kilometres

The ratio of road distance to footpath distance = 3:2

So, the total ratio = 3+2 = 5

To determine the work out of road distance

Let x represent the work out of the road distance

x =3/5 * 20

x = 60/5

x = 12 kilometres

Therefore, the distance of road is 12 kilometres

Read more about Ratio

https://brainly.com/question/29754679

#SPJ1


Related Questions

Align the variables in the equations.
2x - 3y = 9
1-6x +9y = -7

Answers

The variables in the equations are aligned as follows:   2x - 3y = 9 and

2x - 3y = 8/3

What does it mean by align a system of linear equation?

When we talk about aligning a system of linear equations, we mean rearranging the equations so that the variables are in the same order and have the same coefficients. This is done to make it easier to apply methods for solving systems of equations, such as substitution or elimination.

In a system of linear equations, each equation typically involves two or more variables. The variables may have different coefficients in each equation, and they may appear in a different order. Aligning the system involves rearranging the equations in a way that puts the variables in the same order, with the same coefficients.

Align the variables in given the system of linear equations :

To align the variables in the given equations, we need to rearrange the second equation so that the coefficients of  and  are the same as in the first equation.

To align the variables in the equations, we need to rearrange the terms so that the x, y, and constant terms are all grouped together.

Starting with the first equation:

2x - 3y = 9

We can rearrange this as:

2x = 3y + 9

Now we can divide both sides by 2 to get x by itself:

x = (3/2)y + 4.5

Now let's move on to the second equation:

1-6x +9y = -7

We can rearrange this as:

-6x + 9y = -8

Next, we'll divide both sides by -3 to get x by itself:

2x - 3y = 8/3

Now both equations are in the form of:

ax + by = c

where a, b, and c are constants. The aligned equations are:

2x - 3y = 9

2x - 3y = 8/3

To know more about variables visit :

brainly.com/question/29583350

#SPJ1

Mai poured 2. 6 liters of water into a partially filled pitcher. The pitcher then contained 10. 4 liters. How much water did the pitcher contain before Mai added more water?

Answers

The amount of water that the pitcher contained before Mai added more water was 7.8 liters.

Let x be the amount of water the pitcher contained before Mai added more water.

When Mai poured 2.6 liters of water, the total amount of water in the pitcher became x + 2.6 liters.

According to the problem, the pitcher then contained 10.4 liters of water. Therefore, we can write:

x + 2.6 = 10.4

Subtracting 2.6 from both sides, we get:

x = 7.8

Therefore, the pitcher contained 7.8 liters of water before Mai added more water.

To learn more about liters Click here:
brainly.com/question/14780251

#SPJ4

which of the following conditions must be met to conduct a two-proportion significance test? the populations are independent. the probabilities of success multiplied by the sample sizes are greater than or equal to 10 and the probabilities of failure multiplied by the sample sizes are greater than or equal to 10 for each population. the sample sizes are greater than 30.

Answers

The following conditions must be met to conduct a two-proportion significance test:

the populations are independent, the probabilities of success multiplied by the sample sizes are greater than or equal to 10 and the probabilities of failure multiplied by the sample sizes are greater than or equal to 10 for each population, and the sample sizes are greater than 30.

The two-proportion significance test is a hypothesis test that compares the proportions of two independent populations.

To conduct the two-proportion significance test, the following conditions must be met:

Populations must be independent.

Sample sizes are greater than 30.

The probabilities of success multiplied by the sample sizes are greater than or equal to 10 and the probabilities of failure multiplied by the sample sizes are greater than or equal to 10 for each population.

The sample size should be large enough so that the sampling distribution of the sample proportion is nearly normal. The sample sizes should be large enough so that the central limit theorem can be applied.

In short, to conduct a two-proportion significance test, the populations must be independent, the probabilities of success multiplied by the sample sizes are greater than or equal to 10 and the probabilities of failure multiplied by the sample sizes are greater than or equal to 10 for each population, and the sample sizes are greater than 30.


To know more about two-proportion significance test refer here:

https://brainly.com/question/30255520#

#SPJ11

GIVING BRAINLIEST FOR THE CORRECT ANSWER (i need a proof that what you’re saying is right bc ppl are giving me the wrong answers)

Answers

Answer:

x [tex]\geq[/tex]2

Step-by-step explanation:

Since the arrow is pointing to the right, we know that it is greater than two. We also know that it could be equal to 2 because the dot is filled in on the number line. So, the answer is x is greater than or equal to 2.

The triangles are similar. Find the value of x.

Answers

Since the triangles are similar, the value of x is equal to: C. 18 units.

What are the properties of similar triangles?

In Mathematics, two (2) triangles are said to be similar when the ratio of their corresponding side lengths are equal and their corresponding angles are congruent.

By applying the properties of similar triangles, we have the following ratio of corresponding side lengths;

AC/RS = AB/RT

By substituting the given side lengths into the above equation, we have the following:

x/24 = 24/32

By cross-multiplying, we have the following;

32x = 24(24)

32x = 576

x = 576/32

x = 18 units.

Read more on triangle here: brainly.com/question/22891315

#SPJ1

Missing information:

The question is incomplete and the complete question is shown in the attached picture.

you roll a dice with 6 sides what is the probability of....... write your answer as a fraction....roll a 5,roll a 6,roll a c or 4,roll an odd number,roll an even nuber,roll a number greater than 3?,roll an even number less that 5?,roll a multiple of 2(2,4,6),roll a factor of 6(6,4,2,1).

Answers

Roll a 5: There is only one way to roll a 5 out of six possible outcomes, so the probability of rolling a 5 is 1/6.

Roll a 6: Similarly, there is only one way to roll a 6 out of six possible outcomes, so the probability of rolling a 6 is 1/6.

Roll a c or 4: There are two ways to roll a 4 (rolling a 4 or rolling a 3) and one way to roll a 3, so there are three ways to roll a 4 or c out of six possible outcomes. Therefore, the probability of rolling a 4 or c is 3/6, simplifying it to 1/2.

Roll an odd number: There are three odd numbers (1, 3, 5) out of six possible outcomes, so the probability of rolling an odd number is 3/6, simplifying to 1/2.

Roll an even number: There are three even numbers (2, 4, 6) out of six possible outcomes, so the probability of rolling an exact number is 3/6 or 1/2.

Roll a number greater than 3: There are three numbers greater than 3 (4, 5, 6) out of six possible outcomes, so the probability of rolling a number greater than 3 is 3/6, which simplifies to 1/2.

Roll an even number less than 5: There is only one number less than 5 (2) out of six possible outcomes, so the probability of rolling an actual number less than 5 is 1/6.

Roll a multiple of 2 (2, 4, 6): There are three multiples of 2 out of six possible outcomes, so the probability of rolling a multiple of 2 is 3/6, simplifying to 1/2.

Roll a factor of 6 (1, 2, 3, 6): There are four factors of 6 out of six possible outcomes, so the probability of rolling a factor of 6 is 4/6, which simplifies to 2/3.

So the probabilities for each event expressed as fractions are:

Roll a 5: 1/6

Roll a 6: 1/6

Roll a 4 or c: 1/2

Roll an odd number: 1/2

Roll an even number: 1/2

Roll a number greater than 3: 1/2

Roll an even number less than 5: 1/6

Roll a multiple of 2: 1/2

Roll a factor of 6: 2/3

Help me with these please!!

Answers

The angle ABD is 35 degrees, AC is 20 units long, and AB is 29 units long.

What in mathematics is an angle?

An angle is created by combining two rays (half-lines) that have a common terminal. The angle's vertex is the latter, while the rays are alternately referred to as the angle's legs and its arms.

Triangle ABD's angle ABC is one of its outside angles, making it equal to the sum of the opposing interior angles.

Angle ABC = Angle ABD + Angle ACD

replacing the specified values:

110° = Angle ABD + 75°

Simplifying:

Angle ABD = 110° - 75°

Angle ABD = 35°

Due of their shared angles, the two triangles are comparable. This fact can be used to establish a ratio between the corresponding sides:

AC / CD = AB / BD

replacing the specified values:

AC / 10 = 16 / 8

Simplifying:

AC = 20

To know more about angle visit:-

brainly.com/question/28451077

#SPJ1

Complete Question:

Find the angle ∠ABD jn the given figure

the stopping distance s of a car varies directly as the square of its speed v. if a car traveling at 40 mph requires 80 ft to stop, find the stopping

Answers

If a car traveling at 40 mph requires 80 feet to stop, the stopping distance S of a car varies directly as the square of its speed v and is equal to 180 feet.


Given, the stopping distance S of a car varies directly as the square of its speed v. So the relation can be represented as,

S ∝ v2

Here, the constant of proportionality is k.

S = kv2 ——— (1)

Given, when the speed v = 40 mph, stopping distance s = 80 feet.

Therefore, from equation (1), we have

80 = k × 402

k = 80/1600

k = 0.05

Hence, the relation between the stopping distance S and the speed v of the car can be given as

S = 0.05v2

To find the stopping distance S of the car at speed v = 60 mph, substitute v = 60 in the above equation.

S = 0.05 × 602

S = 0.05 × 3600

S = 180 feet

Therefore, the stopping distance of a car traveling at 60 mph would be 180 feet.


To know more about stopping distance refer here:

https://brainly.com/question/24254597#

#SPJ11

what is the probability that the first question she gets right is question number 4? group of answer choices

Answers

The probability that the first question she gets right is question number 4, is 0.1054.

Number of options there are for a single query = 4

P(guessing correct answer for a single question) = 1/4

P(guessing correct answer for a single question) = 0.25

Probability of getting correct answer P(correct) = 0.25

Probability of getting wrong answer P(wrong) = 1 - Probability of getting correct answer

Probability of getting wrong answer P(wrong) = 1 - 0.25

Probability of getting wrong answer P(wrong) = 0.75

So, the probability that the first question she gets right is question number 4 = Probability of getting 1st question wrong × Probability of getting 2nd question wrong × Probability of getting 3rd question wrong × Probability of getting 4th question right

The probability that the first question she gets right is question number 4 = 0.75 × 0.75 × 0.75 × 0.25

The probability that the first question she gets right is question number 4 = 0.1055

To learn more about probability link is here

brainly.com/question/31120123

#SPJ4

The complete question is:

In a multiple choice exam, there are 5 questions and 4 choices for each question (a, b, c, d). Nancy has not studied for the exam at all and decides to randomly guess the answers. (Round your answers to four decimal places.)

What is the probability that the first question she gets right is question number 4?

Triangle ABC is shown below. Describe how we could use circles to determine whether this is an equilateral triangle.

Answers

In Triangle ABC, we can draw a circle around each vertex of the triangle.

If the radii of all three circles are equal, then Triangle ABC is an equilateral triangle.

What is radii?

Radii is the plural form of radius. It is half of the diameter and the length of the radius is used to calculate the area and circumference of the circle.

To determine whether a triangle is an equilateral triangle, we can use circles.

In this method, we draw three circles, one around each vertex of the triangle.

We then measure the radii of the circles, ensuring that they are all equal. If the radii of the three circles are equal, then the triangle is an equilateral triangle.

In Triangle ABC, we can draw a circle around each vertex of the triangle. Then, we could use a measuring tool to measure the radii of each circle. If the radii of all three circles are equal, then Triangle ABC is an equilateral triangle.

This method of using circles to determine whether a triangle is an equilateral triangle is simple and efficient. It does not require any complex calculations, and it is easy to understand.

For more questions related to equilateral triangle

https://brainly.com/question/17264112

#SPJ1

8x 2 + [ 3x3-8] = with explanation pls and its due in six minutes

Answers

Answer: 16x+3x^3−8

Please mark me brainliest :)

Noah was at home. He got on his bike and rode to his friends

Answers

Answer:

what's your exact question

Answer:

can u pls type the full question

Proportions

Two plus x divided by twelve equals one dived by three. Solve for x.

Answers

Two plus x divided by twelve equals one divided by three

Case 1 :

Rewrite into numbers : 2 + x /12 = 1/3

-> x/12 = 1/3 - 2 = -5/3

-> x = -5/3 x 12 = -20

Case 2 :

Rewrite into numbers : (2 + x)/12 = 1/3

-> 2 + x = 1/3 x 12 = 4

-> x = 4 - 2 = 2

i dont know if you meant it the right way or the wrong way but ill just put them both

x=2

Step-by-step explanation:

(2+x)/12=1/3

3(2+x)=12

2+x=4

x=4-2

x=2

The point on the graph represents Ann's location. She is using a metal detector on the beach to see what she can find. Each unit on the graph represents 2 feet. A pile of bottle caps is located at (4, -10). Find the length of the most direct path between Ann and the pile of bottle caps. Round to the nearest whole number.

Answers

Answer:

30 feet

Step-by-step explanation:

Coordinates of Ann: (-4,3)

Coordinates of bottle caps: (4,-10)

Distance from Ann to bottle caps can be found out using the distance formula:
[tex]x_2=4, x_1=-4\\y_2=-3,y_1=-10\\Distance=\sqrt{(x_{2}-x_{1})^2 + (y_{2}-y_{1})^2 } \\=\sqrt{(4-(-4))^2 + ((-10)-3)^2} \\=15.26\\15\text{ is the answer}[/tex]

solve for x (Please leave an explanation)

Answers

The value of x for the angle 100 + x under the top parallel line is equal to -10.

What are angles formed by a pair of parallel lines cut by a transversal line?

When a transversal line intersects a pair of parallel lines, several angles are formed which includes: Corresponding angles, vertical angles, and alternate angles.

The angle 100 + x formed by the top parallel line and the transversal line perpendicular to to it is equal to 90° so we can solve for the value of x as follows:

100 + x = 90

subtract 100 from both sides

x = 90 - 100

x = -10.

Therefore, the value of x for the angle 100 + x under the top parallel line is equal to -10.

Know more about angles here:https://brainly.com/question/24607467

#SPJ1

How to turn 0. 1212121212 into a simplified fraction

Answers

Answer:

  4/33

Step-by-step explanation:

You want to write 0.1212...(repeating) as a simplified fraction.

Repeating decimal

A repeating decimal beginning at the decimal point can be made into a fraction by expressing the repeating digits over an equal number of 9s.

Here, there are 2 repeating digits, so the basic fraction is ...

  12/99

This can be reduced by removing a factor of 3 from numerator and denominator:

  [tex]0.\overline{12}=\dfrac{12}{99}=\boxed{\dfrac{4}{33}}[/tex]

__

Additional comment

Formally, you can multiply any repeating decimal by 10 to the power of the number of repeating digits, then subtract the original number. This gives the numerator of the fraction. The denominator is that power of 10 less 1.

  0.1212... = (12.1212... - 0.1212...)/(10^2 -1) = 12/99

Doing this multiplication and subtraction also works for numbers where the repeating digits don't start at the decimal point. Finding a common factor with 99...9 may not be easy.

You can also approach this by writing the number as a continued fraction. The basic form is ...

  [tex]x=a+\cfrac{1}{b+\cfrac{1}{c+\cdots}}[/tex]

where 'a' is the integer part of the original number, and b, c, and so on are the integer parts of the inverse of the remaining fractional part. The attachment shows how this works for the fraction in the problem statement.

A calculator cannot actually represent a repeating decimal exactly, so error creeps in and may eventually become significant.

the radius of a circle is changing at .5 cm/sec. find the rate of change of the area when the radius is 4 cm.

Answers

The rate of change of the area of a circle when the radius is 4 cm is 4π cm2/sec.

This can be calculated using the formula for the area of a circle (A = πr2) and the chain rule for derivatives. The chain rule states that when the radius (r) changes, the area of a circle (A) is equal to 2πr times the rate of change of the radius (dr/dt).

Therefore, the rate of change of the area of a circle when the radius is 4 cm is equal to 2π(4 cm) × (0.5 cm/sec) = 4π cm2/sec.

Note that if the rate of change of the radius were different, the rate of change of the area would also be different. This formula can be used to calculate the rate of change of the area of a circle at any given radius, as long as the rate of change of the radius is known.

For more such questions on Rate of change of area.

https://brainly.com/question/30435679#

#SPJ11

if the matrix product a1b is known, how could you calculate b1a without necessarily knowing what a and b are?

Answers

We can calculate its product by taking the dot product of each row of B1A and each column of A1B. In this way, we can calculate B1A without knowing the values of A and B.

The matrix product of two matrices, A and B, is defined as the matrix C, where C = AB. To calculate the product of two matrices, we must take the dot product of each row of A and each column of B. If we are given a matrix product A1B, then we can calculate B1A without necessarily knowing what A and B are.

To do so, we must first invert the matrix A1B. We can do this by solving a system of equations. We can set up this system of equations by treating the entries of A1B as the coefficients in a system of equations, and solving for the entries of B1A. Once we have found the inverse, we can calculate the matrix B1A.

Finally, once we have the matrix B1A, we can calculate its product by taking the dot product of each row of B1A and each column of A1B. In this way, we can calculate B1A without knowing the values of A and B.

See more about matrix at: https://brainly.com/question/14414517

#SPJ11

Jason has a block of clay that is made up of two rectangular pieces of
different colors. Find the volume of the block of clay.
The measurements of the two clay blocks are:
6 cm
4 cm
3 cm
5 cm

Answers

Answer: 132 cubic centimeters

Step-by-step explanation:

To find the volume of the block of clay, we need to add the volumes of the two rectangular pieces.

The volume of a rectangular solid can be found by multiplying its length, width, and height. Let's call the first rectangular piece A and the second rectangular piece B. Then the dimensions of A are 6 cm (length), 4 cm (width), and 3 cm (height), and the dimensions of B are 5 cm (length), 4 cm (width), and 3 cm (height).

The volume of A is:

Volume of A = length x width x height = 6 cm x 4 cm x 3 cm = 72 cubic centimeters

The volume of B is:

Volume of B = length x width x height = 5 cm x 4 cm x 3 cm = 60 cubic centimeters

So the total volume of the block of clay is:

Volume of block = Volume of A + Volume of B = 72 cubic centimeters + 60 cubic centimeters = 132 cubic centimeters

Therefore, the volume of the block of clay is 132 cubic centimeters.

suppose there is $600 in the account with an annual interest rate of 4%. after how many years will the amount triple?

Answers

it will take approximately 22.56 years for the amount to triple.

The given information for this problem is that there is an initial investment of $600 in an account with an annual interest rate of 4%. The task is to determine after how many years the amount will triple.Using the compound interest formula, we can find the amount in the account after t years:A = P(1 + r/n)nt Where,A = final amount in the account, P = initial amount in the account r = annual interest rate ,n = number of times the interest is compounded per year ,t = time in years.

From the problem statement, we know that the initial amount, P, is $600 and the annual interest rate, r, is 4%. Let's assume that the interest is compounded annually, i.e., n = 1.Substituting these values in the formula, we get:A = $600(1 + 0.04/1)1t Simplifying this expression,A = $600(1.04)t.

Taking the ratio of the final amount to the initial amount, we get: 3P = $600 × 3 = $1800. Therefore,A/P = 3 = (1.04)t.Dividing both sides by P, we get:3 = (1.04)t ln(3) = ln(1.04)t. Using the logarithmic property, we can bring down the exponent to the front:ln(3) / ln(1.04) = t Using a calculator, we get ≈ 22.56. Therefore, it will take approximately 22.56 years for the amount to triple.

To know more about compound interest, click here:

https://brainly.com/question/22621039

#SPJ11

The supplement of an angle is 30 more than twice its complement. What is the measure of the
angle?

Answers

Answer: 30

180 - x = 180 - 2x + 30

x = 30

Answer:

The measure of the unknown angle is 30°.

Step-by-step explanation:

Let the measure of the unknown angle be x°.

Supplementary angles are two angles whose measures sum to 180°.

Complementary angles are two angles whose measures sum to 90°.

Therefore, the supplement of x° is (180 - x)°, and its complement is (90 - x)°.

Given that the supplement is 30° more than twice its complement:

(180 - x)° = 2(90 - x)° + 30°

To find the measure of the angle, solve the equation:

⇒ (180 - x)° = (180 - 2x)° + 30°

⇒ 180° - x° = 180° - 2x° + 30°

⇒ 180° - x° = 210° - 2x°

⇒ 180° - x° + 2x° = 210° - 2x° + 2x°

⇒ 180° + x° = 210°

⇒ 180° + x° - 180° = 210° - 180°

⇒ x° = 30°

Therefore, the measure of the unknown angle is 30°.

To make cleaning easier, a rectangular horse trough will be lined with plastic. The trough is 40 inches long, 14 inches wide, and 24 inches deep. How many square inches of plastic are needed to line the trough? Count only the trough's five faces. A net containing 5 rectangles. Two rectangles have length of 40 inches and width of 14 inches. Two rectangles have length of 14 inches and width of 24 inches. One rectangle has length of 40 inches and width of 24 inches.

Answers

Using the area formula for the rectangle, we can find that 2752 in² of plastic is needed to line the trough.

Define area?

To determine the area a rectangle occupies within its perimeter, apply the formula for calculating a rectangle's area. Multiplying the length by the width yields the area of a rectangle (breadth).

As a result, the area of a rectangle with the length and breadth l and w, respectively, is calculated as follows. L × W = the rectangle's area. Hence, the area of a rectangle is equal to (length width).

Now in the given question,

We have 5 faces of the cuboid.

Now to find the total area of the required space we have to find the area of all the rectangles.

Area of rectangle with dimensions, l = 40inches and b = 14 inches.

Area = l × b

= 40 × 14

= 560in²

Now there are 2 rectangles with the same dimensions, so the total area = 560 + 560 = 1120in².

Now area of rectangles with dimensions, l = 14 inches and b = 24 inches.

Area = l × b

= 14 × 24

= 336in².

There are 2 rectangles with the same dimensions, so area = 336 + 336 = 672in².

Area of the final rectangle = l × b

= 40 × 24

= 960in².

So, the total required area = 1120 + 672 + 960 = 2752in².

To know more about area of rectangles, visit:

https://brainly.com/question/16309520

#SPJ1

Simon is making a frame for a photo that measures 6 inches by 8 inches. He wants the frame to be the same width all the way around and the total area of the frame and photo to be 120 square inches. Which quadratic equation represents the area of the picture and frame?​

Answers

Answer: A

Step-by-step explanation:

which value of n makes the equation true
-[tex]\frac{1}{2}n=-8[/tex]

Answers

Answer:

16

Step-by-step explanation:

When we divide the entire equation by 1/-2 to get rid of the coefficient of n we get n = 16.

9 km
7 km
3 km
3 km
3 km
2 km
8 km
9 km
3 km
7 km

Answers

Answer: what do you mean? I need more info-

Step-by-step explanation:

I can answer it with more info :)

if the odds on a bet are 16:1 against, what is the probability of winning? express your answer as a fraction.

Answers

The probability of winning is 1/17, which can also be expressed as a decimal (approximately 0.059) or as a percentage (approximately 5.9%).

The odds on a bet represent the ratio of the probability of winning to the probability of losing. In this case, the odds are 16:1 against winning, which means that the probability of winning is 1 out of 16.

To express this probability as a fraction, we can use the formula:

Probability of winning = 1 / (odds + 1)

Plugging in the given odds, we get:

Probability of winning = 1 / (16 + 1)

Probability of winning = 1/17

In this case, the odds of 16:1 against winning correspond to a probability of 1/17, which represents the chance of winning the bet.

To learn more about probability click on,

https://brainly.com/question/29111123

#SPJ4

Determine whether segment lengths form a triangle. If so, classify the triangle as acute, right or obtuse.

1. 10, 7, sqrt(658)

Answers

Answer:

it is a triangle bc it has angles of points

Step-by-step explanation:

Help please, Which value of x satisfies the equation 7/3(x+9/28)=20

Answers

Answer:

Step-by-step explanation:

[tex]\frac{7}{3} (x+\frac{9}{28} )=20[/tex]    

[tex]7 (x+\frac{9}{28} )=60[/tex]     (multiplied both sides by 3)

[tex]x+\frac{9}{28} =\frac{60}{7}[/tex]          (divided both sides by 7)

[tex]x=\frac{60}{7}-\frac{9}{28}=\frac{240}{28}-\frac{9}{28}=\frac{231}{28} =8.25[/tex]     (subtracted [tex]\frac{9}{28}[/tex] both sides and solved)

a children's liquid medicine contains 100 mg of the active ingredient in 5 ml . if a child should receive 300 mg of the active ingredient, how many milliliters of the medicine should the child be given? for the purposes of this question, assume that these numbers are exact.

Answers

The child should be given 15 ml of the medicine to receive 300 mg of the active ingredient.

The given problem requires us to determine the number of milliliters of a liquid medicine that a child should receive in order to obtain a specific dosage of the active ingredient. We are given that the medicine contains 100 mg of the active ingredient in 5 ml.

The child needs to receive 300 mg of the active ingredient, and there are 100 mg of the active ingredient in 5 ml of the medicine. Therefore, the child should be given:

[tex]\frac{300 mg}{100mg/5ml} = \frac{300\text{ mg} \times 5\text{ ml}}{100\text{ mg}} = 15\text{ ml}$$[/tex]

So the child should be given 15 ml of the medicine to receive 300 mg of the active ingredient.

Learn more about Liquid Measurements:

https://brainly.com/question/7744911
#SPJ4

PLEASE HELP An elevator goes down 3 floors, up 5 floors, and finally down 7 floors. Which expression represents the total number of floors the elevator traveled?

*
A | -3 | + | 5 | + | -7 |
B | -3 | - | 5 | + | -7 |
C ( -3 ) + 5 + ( -7 )
D 3 - 5 + 7

Answers

Answer:

B is the answer

Step-by-step explanation:

Other Questions
Solve the following quadratic-like equation.[tex]y^\frac{1}{2} -6y^\frac{1}{4} +8=0[/tex] one cycle of the power dissipated by a resistor ( ) is given by this periodic signal repeats in both directions of time. what is the amplitude of the pwm voltage signal applied across the 500- resistor what did the study by cahill, lancaster, and green (2010) determine about smoking cessation interventions matched to stage theories as compared to uniform interventions consistent with continuum theories? what is the correct equation for cellular respiration showing the reactants on the left of the equal sign and products on the right of the equal sign? Which of the following statements correctly characterizes changes in farming and agriculture land during the period between 1870 and 1900? a. The number of farms, as well as the number of acres under cultivation rose b. The number of farms, as well as the number of eres under cultivation fell. c. The number of farms rose, but the number of atres under cultivation fell d. The number of farms fel but the number of ander cultivation rose which of the following are true of asexual reproduction? choose all that apply. the offspring are genetically different than the parents. there is only one parent. the offspring are genetically identical to the parent. there are two parents. t a fixed temperature and number of moles, the initial volume and pressure of a helium gas sample are 153 ml and 433 torr, respectively. what is the final volume in ml, if the final pressure is 67.1 torr? Nathaniel would like to establish a trust fund that will provide R380 000 a year, forever, for his descendants. The trust fund will be invested very conservatively, so the expected rate of return is only 6,45%. How much money must he deposit today, to fund this gift for his descendants? Geography is the study of areal differentiation." Discuss with examples when many buyers and sellers freely enter and exit a market having similar, yet differentiated products, it is called . multiple choice question. perfect competition oligopoly monopolistic competition monopoly An appliance store decreases the price of a 19-in. television set 28% to a sale price of $435.60.What was the original price? OA. cacheC. URLAttemptsA browser stores images and page elements in a on your device.Save AnswerOB. vaultD. cookie the transition of Russia to communism following the October revolution occurred during which of these events.A. cold war B. world war 2 C. Spanish American war D. world war 1 what has been the outcome of the effort to preserve the native american lakota language, which today is spoken by about 25,000 people in the united states? vhat is the volume of the composite figures 104 ft120 ft150 ft160 ft calculate the density (in grams per milliliter) for a glass marble with a volume of 7.94 ml and a mass of 15.36 g. the following frequency distribution displays the weekly sales of a certain brand of television at an electronics store. number sold frequency 01-05 12 06-10 5 11-15 5 16-20 5 21-25 25 how many weeks of data are included in this frequency distribution? solve the inequality -8x < 32 should it be reveresed impairment losses on property, plant and equipment are normally reported on the income statement as: a rising parcel of unstable air a rising parcel of unstable air can rise well into the mesosphere. cannot rise very far above the tropopause. can eventually escape into space. will not be slowed by entrainment.